ϕ4ϕ4\phi^{4} Propagador - Diagrama de Feynman: vértice interno que regresa a sí mismo

En todo lo que sigue me ocuparé de todo sin masa.

El propagador libre y sin masa ( L = d 4 X ( ϕ ( X ) ) 2 ) supuestamente está dada por GRAMO 0 ( X , y ) = C ( X y ) 2 , donde creo C = 1 4 π 2 .

Estoy tratando de calcular el propagador en ϕ 4 -teoría, específicamente la contribución debido a este diagrama:ingrese la descripción de la imagen aquí

Usando las reglas de Feynman en el espacio de posición, creo que debería obtener una contribución de la forma:

C ( X 1 , X 2 ) = i λ d 4 tu   GRAMO 0 ( X 1 , tu ) GRAMO 0 ( tu , tu ) GRAMO 0 ( tu , X 2 )

Sin embargo, aquí está mi pregunta: ¿por qué obtengo GRAMO 0 ( tu , tu ) = C ( tu tu ) 2 = tu norte d mi F i norte mi d ? No hay manera que pueda ver para evaluar esta integral.

¿Cómo trato con esto? ¿Tal vez tengo el orden incorrecto de las variables? Soy nuevo en este tipo de cálculos.

Respuestas (2)

Para definir un ϕ 4 teoría sin masa, no puede simplemente comenzar desde un lagrangiano

L = 1 2 ( ϕ ) 2 gramo 0 4 ! ϕ 4 ,
separar un término cinético L 0 = 1 2 ( ϕ ) 2 y hacer la teoría de la perturbación alrededor gramo 0 = 0 . La razón es que, incluso en el orden más bajo de la teoría de la perturbación, la ϕ 4 término dará lugar a diagramas divergentes como el que acabas de calcular, que son formalmente infinitos. Para tratar tales diagramas, necesita:

  1. Para introducir un regulador, como un corte UV Λ en los propagadores.
  2. Imponer algunas (en este caso, tres) condiciones de finitud a las funciones de Green.
  3. Para expresar todas las cantidades en términos de los parámetros (finitos) introducidos en el paso 2.

El procedimiento funcionará solo si comienza desde un lagrangiano renormalizable como:

L = 1 2 ( ϕ ) 2 metro 0 2 2 ϕ 2 gramo 0 4 ! ϕ 4 .

El propósito del Paso 1 es dar sentido a cantidades divergentes como GRAMO 0 ( X X ) . Una manera simple que funciona para el ϕ 4 La teoría es la regularización de Pauli-Villars:

GRAMO ~ 0 ( pag ) = 1 pag 2 metro 0 2 + i ε GRAMO ~ 0 ( pag ) Λ = 1 pag 2 metro 0 2 + ( pag 2 ) 2 Λ 2 + ( pag 2 ) 3 Λ 4 + i ε .

En el paso 2 podemos al mismo tiempo hacer la teoría sin masa y eliminar el diagrama divergente que ha calculado. Simplemente requerimos que 1 :

GRAMO ~ ( pag 2 = 0 ) 1 = 0.
Esto asegura que la masa física de las partículas descritas por la teoría es cero, y si calculas el propagador a primer orden en gramo 0 , también encontrará que el valor constante del diagrama divergente se cancela exactamente por la masa desnuda metro 0 2 .


1 En realidad, a primer orden en gramo 0 , esta condición es suficiente para renormalizar la teoría.

Si nunca ha visto nada relacionado con la renormalización, esto probablemente no tenga mucho sentido, lo siento. En ese caso, la esencia es que su cálculo es formalmente correcto en el sentido de que da un resultado indefinido. Esto necesita ser tratado con técnicas de regularización y renormalización.
No sabía que no se puede tener sin masa ϕ 4 del lagrangiano, quizás por eso no lo he visto en clase. Dado que la renormalización de masa es multiplicativa, el diagrama de renacuajo debe ser cero para que la teoría no tenga masa. ¡Gracias por esta idea! :-)
@Martin Ueding ¡Me alegra ayudar! También estoy aprendiendo estas cosas en este momento y mi maestro se centró en el esquema de "corte duro" que describí anteriormente para el ϕ 4 teoría. En este esquema, la condición de polo da, a primer orden en gramo 0 :
metro 0 2 + gramo 0 2 Δ ( 0 ; Λ ) = 0 ,
dónde Δ ( 0 ; Λ ) es el propagador regulado en X = 0 . Forzar metro 0 2 = 0 echa a perder la renormalizabilidad de la teoría incluso a primer orden, porque hace imposible cancelar un término divergente como Δ ( 0 ; Λ ) en el propagador.

Acabo de intentar calcular este diagrama en el espacio de momento y creo que da una contribución cero en el caso sin masa. Básicamente esto se debe a que dentro del ciclo obtendrás una integral como

d 4 k ( 2 π ) 4 i k 2 metro 2 + i ϵ ,
dónde metro = 0 . Un profesor dijo que esta integral dará cero porque para metro = 0 , no hay una escala de masa/energía que esta integral de bucle pueda probar. Recuerdo que la explicación no me satisfizo en su momento, lamentablemente no tengo una mejor en este momento.

Luego también con bucles, no es raro que den infinitos. Esto conducirá a la regularización y eventualmente a la renormalización que se siente como la caja de Pandora. Su integral parece divergir ya que hay ocho potencias de tu en el numerador (de la medida de integración) pero sólo seis potencias en el denominador (de las funciones de Green). Todavía podría ser infinito. Luego tienes que ir al espacio de momento y usar la regularización dimensional.

Puedo darte una derivación del espacio de momento de este diagrama de renacuajo con regularización dimensional que hice hace un par de años como tarea en la clase de QFT que tomé. Puede descargar el PDF original y revisado para leerlo completo. Este es el extracto:

ingrese la descripción de la imagen aquí

ingrese la descripción de la imagen aquí

El resultado final es proporcional a metro . Configuración metro = 0 hará que esta cosa desaparezca, por lo que el resultado parece ser cero en la teoría sin masa.

Entonces, en cierto sentido, un vértice interno que vuelve a sí mismo, ¿se ve como dos vértices internos? (ya que tenemos que integrar dos veces, como dices)
Espera no. Lo que escribí ya no tiene sentido para mí. Pensando en el espacio de impulso, solo hay un impulso de bucle libre. Y solo hay una integral. De todos modos, ¿de dónde sacaste ese propagador? ¿No debería ser 0 | ϕ ( X ) ϕ ( y ) | 0 = d 3 pag ( 2 π ) 3 1 2 mi pag Exp ( i pag ( X y ) ) ? cf. (290) .
Ya veo, está bien, estaba pensando que solo debería haber una sola integral también. Aquí hay un enlace que discute este propagador physics.stackexchange.com/q/101886 , también hay un poco sobre el propagador que he usado en el QFT de Birrell y Davies en Curved Spacetime. Creo que el propagador que ha escrito se reduce a esta forma en el caso sin masa de alguna manera.
Al revisar mis propias notas de QFT, descubrí que he calculado este diagrama. Parece desaparecer en el caso sin masa, al menos cuando uno realiza el cálculo con regularización dimensional. En la versión del propagador que tienes, todavía no veo cómo se debe extraer un resultado sensato.